Đến nội dung

quangnghia nội dung

Có 389 mục bởi quangnghia (Tìm giới hạn từ 24-04-2020)



Sắp theo                Sắp xếp  

#617553 Cho a,b,c la các số thực dương thỏa mãn a+b+c=3CMR....

Đã gửi bởi quangnghia on 28-02-2016 - 22:13 trong Bất đẳng thức và cực trị

Cho a,b,c la các số thực dương thỏa mãn a+b+c=3CMR  $\frac{a+1}{b^{2}+1} + \frac{b+1}{c^2+1{}} + \frac{c+1}{a^2+1{}} \geq 3$

Ta có $\frac{a+1}{b^{2}+1}=a+1-\frac{b^{2}(a+1)}{b^{2}+1}\geq a+1-\frac{b(a+1)}{2}\geq a-\frac{b}{2}+1-\frac{ab}{2}$

Thực hiện tương tự ta thu được:

$\Rightarrow \sum \frac{a+1}{b^{2}+1}\geq \frac{a+b+c}{2}+3-\frac{1}{2}(ab+bc+ca)\geq \frac{3}{2}+3-\frac{1}{2}.3\geq 3$




#616290 Viết phương trình đường tròn nội tiếp tam giác ABC

Đã gửi bởi quangnghia on 21-02-2016 - 19:57 trong Phương pháp tọa độ trong mặt phẳng

Viết phương trình đường tròn nội tiếp tam giác ABC biết phương trình 3 cạnh là 3x+4y-6=0, 4x+3y-1=0, y=0

Do bạn không nói rõ nên mình giả sữ phương trình AB, BC, CA lần lượt theo thứ tự $3x+4y-6=0, 4x+3y-1=0, y=0$

Ta giải ra $A(2,0), B(-2,3), C(\frac{1}{4},0)$

Độ dài $AB=5, BC=\frac{15}{4},AC=\frac{7}{4}$

Gọi AI là phân giác góc A (I thuộc BC)

$\frac{BI}{IC}=\frac{AB}{AC}=\frac{20}{7}, BI+IC=BC =\frac{15}{4}$

$\Rightarrow BI=\frac{25}{9}$

$\Rightarrow \frac{BI}{BC}=\frac{20}{27}\Rightarrow \overrightarrow{BI}=\frac{20}{27}\overrightarrow{BC}$

$I(i,\frac{1-4i}{3})$

$\overrightarrow{BI}=(i+2,\frac{-8-4i}{3})$

Từ đây giải ra i, ta có tọa độ I. Ta viết phương trình AI. Rồi từ AI tìm 1 điểm K sao cho khoảng cách từ K tới AB bằng khoảng cách từ K đến BC. Giải ra tọa độ K. Bài toán kết thúc khi có tọa độ K và bán kính (khoảng cách từ K đến BC)




#616272 Chứng minh rằng M di chuyển trên một đường thẳng cố định khi C di chuyển trên...

Đã gửi bởi quangnghia on 21-02-2016 - 18:37 trong Hình học

Cho đường tròn (O), đường kính AB = 2R. Trên (O) lấy điểm C sao cho AC > BC. Trên đoạn thẳng OB lấy điểm I là một điểm cố định. Qua I dựng đường thẳng d vuông góc với AB; đường thẳng d cắt BC tại E, cắt AC tại F. Gọi M là tâm đường tròn ngoại tiếp tam giác AEF. Chứng minh rằng khi C di chuyển trên đường tròn (O) thì M luôn di chuyển trên một đường thẳng cố định

Gọi T là giao điểm của $(AEF)$.

Do $ATFE$ nội tiếp nên $ IT.IA=IF.IE$  (1)

Ta có $\Delta AFI\sim\Delta EBI\Rightarrow \frac{IF}{IB} =\frac{AI}{EI}\Rightarrow IF.IB=IA.IB$  (2)

Từ (1)(2) $\Rightarrow IT.AI=IA.IB\Rightarrow IT=\frac{IA.IB}{IA}$

Suy ra T cố định, mà M thuộc trung trực AT nên M di chuyển trên đường thẳng cố định




#615835 Chứng minh $\frac{AB}{AN}$+1=$\f...

Đã gửi bởi quangnghia on 19-02-2016 - 10:23 trong Hình học

cho tam giác ABC có phân giác BD cắt trung tuyến AM tại I, CI cắt AB tại N. Chứng minh $\frac{AB}{AN}$+1=$\frac{2AM}{AI}$

Ta có $\frac{AB}{AN}+1=2\frac{AM}{AI}$

$\Leftrightarrow\frac{AN+NB}{AN}+1=2.\frac{AI+IM}{AI}$

$\Leftrightarrow 1+\frac{NB}{AN}+1=2+2\frac{IM}{AI}$

$\Leftrightarrow \frac{BC}{AC}=2\frac{MC}{AC}$

$\Leftrightarrow BC=2MC$ ( luôn đúng)




#615834 Chứng minh $\frac{AB}{AN}$+1=$\f...

Đã gửi bởi quangnghia on 19-02-2016 - 09:41 trong Hình học

Cho A nằm ngoài (O;R). Kẻ 2 tiếp tuyến AB,AC . Điểm M thuộc cung bc nhỏ (M khác B và C). Tiếp tuyến tại M cắt AB, AC tại E,F. BC cắt OE, OF tại P, Q.

CMR $\frac{PQ}{EF}$ ko đổi khi M thay đổi trên cung BC nhỏ

Ta có $\widehat{EOF}=\widehat{EOM}+\widehat{MOF}=\frac{1}{2}\widehat{BOM}+\frac{1}{2}\widehat{MOC}=\frac{1}{2}\widehat{BOC}=\widehat{BOA}=\widehat{ABC}$

$\Rightarrow \widehat{ABC}=\widehat{EOF}$

$\Rightarrow EBOQ$ nội tiếp mà $\widehat{EBO}=90^{o}\Rightarrow \widehat{EQO}=90^{o}$

$\Rightarrow$ EQ vuông OF.

Tương tự PF vuông EO

$\Rightarrow \Delta OPQ\sim \Delta OFE$

$\Rightarrow \frac{PQ}{EF}=\frac{OP}{OF}=cos\widehat{POF}=cos\frac{\widehat{BOC}}{2}$ là hằng số

điều phải chứng minh




#615733 Chứng minh : a) BD = DI = CD b) IF // BC

Đã gửi bởi quangnghia on 18-02-2016 - 16:48 trong Hình học

Cho tam giác ABC nội tiếp (O), phân giác góc A và góc B cắt nhau ở I, cắt (O) lần lượt tại D và E, DE cắt AC tại F

Chứng minh : a) BD = DI = CD

                       b) IF // BC

a) Ta có do BE là phân giác $\widehat{ABC}$ nên số đo cung AE bằng số đo cung EC. Tương tự số đo cung BD bằng số đo cung DC.

Ta có $\widehat{BID}=\frac{1}{2}(\hat{AE}+\hat{BD})=\frac{1}{2}(\hat{EC}+\hat{DC})=\frac{1}{2}=\hat{ED}=\widehat{EBD}$

Vậy $\widehat{BID}=\widehat{IBD}\Rightarrow \Delta BID$ cân tại D nên $BD=DI$

Mặc khác do $\hat{BD}=\hat{DC}\Rightarrow BD=DC$

Vậy $BD=DI=DC$

b) Ta có $\widehat{AIE}=\frac{1}{2}(\hat{AE}+\hat{BD})=\frac{1}{2}(\hat{AE}+\hat{DC})=\widehat{AFE}$

suy ra tứ giác AEFI nội tiếp

$\Rightarrow \widehat{AFI}=\widehat{AEI}$, mà $\widehat{AEI}=\widehat{ACB}\Rightarrow \widehat{AFI}=\widehat{ACB}$

suy ra IF song song BC




#615702 Giải pt $\sqrt{x+24}+\sqrt{x-16}=10$

Đã gửi bởi quangnghia on 18-02-2016 - 08:50 trong Đại số

Giải pt

$\sqrt{x+24}+\sqrt{x-16}=10$

Đặt $\sqrt{x+24}=a,\sqrt{x-16}=b$

Ta có hệ sau:

$\left\{\begin{matrix} a+b=10 & \\ a^{2}-b^{2}=40 & \end{matrix}\right.$

Từ phương trình đầu ta có $a=10-b$ thay vào phương trình cuối:

$(10-b)^{2}-b^{2}=40\Rightarrow b=3\Rightarrow x=25$




#615687 Cho $\bigtriangleup ABC$ có 3 góc nhọn và AB < AC. Các đườ...

Đã gửi bởi quangnghia on 17-02-2016 - 23:40 trong Hình học

 

Cho $\bigtriangleup ABC$ có 3 góc nhọn và AB < AC. Các đường cao BE và CF cắt nhau tại H ( $E \in AC, F \in AB$ ).

 

1. Chứng minh rằng AE . AC = AF . AB

 

2. Cho$\widehat{BAC}=45^{\circ}.$ Tính $\frac{S_{AEF}}{S_{BCEF}}$

 

3. AH cắt BC tại D. Kẻ đường thằng qua D và song song với EF cắt AB tại M và CF tại N. Chứng minh rằng D là trung điểm của MN.

 

1) Tứ giác BFEC nội tiếp nên $\Rightarrow \widehat{FBE}=\widehat{ECH}$

nên $\Delta AEB\sim \Delta AFC$ (g,g)

$\Rightarrow \frac{AE}{AF}=\frac{AB}{AC}\Rightarrow AE.AC=AF.AB$

2) Do $\widehat{BAC}=90^{o}$ 

$\Rightarrow AC=AF.\sqrt{2}$

Vậy $\Rightarrow \frac{S_{AEF}}{S_{ABC}}=(\frac{AF}{AC})^{2}=\frac{1}{2}$

$\Rightarrow \frac{S_{AFE}}{S_{BFEC}}=1$

3) Ta có $\widehat{DNF}=\widehat{NFE},\widehat{NFE}=\widehat{HAC},\widehat{HAC}=\widehat{DFN}$

$\Rightarrow \widehat{DNF}=\widehat{DFN}$

$\Rightarrow \Delta FDN$ cân tại D $\Rightarrow FD=DN$

Ta có $\widehat{BMD}=\widehat{AFE}, \widehat{AFE}=\widehat{ACB}, \widehat{ACB}=\widehat{BFD}$

$\Rightarrow \widehat{BMD}=\widehat{MFD}$

$\Rightarrow \Delta MDF$ cân tại D

$\Rightarrow MF=FD$

Vậy D là trung điểm MN




#615547 Chứng minh rằng $x^{2010}-1$ chia hết cho $y+1$.

Đã gửi bởi quangnghia on 17-02-2016 - 15:06 trong Đại số

Bài 1: Cho a,b,c là độ dài 3 cạnh hình tam giác. Xác định dạng tam giác để 

$S=\frac{a}{b+c}+\frac{b}{b+c}+\frac{c}{c+a}$ đạt giá trị nhỏ nhất.

Cho $a=b=c$ ta thu được $\frac{3}{2}$. Từ đây ta chứng minh:

$\frac{a}{b+c}+\frac{b}{c+a}+\frac{c}{a+b}\geq \frac{3}{2}$

$\frac{a}{b+c}+1+\frac{b}{c+a}+1+\frac{c}{a+b}+1\geq \frac{9}{2}$

$\Rightarrow \frac{a+b+c}{b+c}+\frac{a+b+c}{a+c}+\frac{a+b+c}{a+b}\geq \frac{9}{2}$

$\Rightarrow (a+b+c)(\frac{1}{a+b}+\frac{1}{b+c}+\frac{1}{c+a})\geq \frac{9}{2}$

$\Rightarrow (a+b+b+c+c+a)(\frac{1}{a+b}+\frac{1}{b+c}+\frac{1}{c+a})\geq 9$           (*)

Ta có $a+b+b+c+c+a\geq 3\sqrt[3]{(a+b)(b+c)(c+a)}$

$\frac{1}{a+b}+\frac{1}{b+c}+\frac{1}{c+a}\geq \frac{3}{\sqrt[3]{(a+b)(b+c)(c+a)}}$

Nhân 2 bất đẳng thức này ta thu được (*)




#615527 $\frac{4}{(a+b)^{3}}+\frac{4}{(b+c)^{3}}+\frac{4}{(c+a)^{...

Đã gửi bởi quangnghia on 17-02-2016 - 12:15 trong Bất đẳng thức và cực trị

Cho a,b,c dương và a+b+c=3 . CMR : $\frac{4}{(a+b)^{3}}+\frac{4}{(b+c)^{3}}+\frac{4}{(c+a)^{3}}\geq \frac{a}{b+c}+\frac{b}{c+a}+\frac{c}{a+b}$

Bất đẳng thức trở thành $\sum \frac{4}{(a+b)^{3}}\geq \sum \frac{3-b-c}{b+c}= \sum \frac{3}{b+c}-3$

Tương đương $\sum \frac{4}{(b+c)^{3}}+3\geq \sum \frac{3}{b+c}$

Ta có $\frac{4}{(a+b)^{3}}+\frac{1}{2}+\frac{1}{2}\geq \frac{3}{a+b}$

$\frac{4}{(b+c)^{3}}+\frac{1}{2}+\frac{1}{2}\geq \frac{3}{b+c}$

$\frac{4}{(c+a)^{3}}+\frac{1}{2}+\frac{1}{2}\geq \frac{3}{c+a}$

Cộng theo vế ta có: $\sum \frac{4}{(b+c)^{3}}+3\geq \sum \frac{3}{b+c}$




#615510 Cho 2 số x,y nguyên dương thỏa mãn: xy = 1. Tính giá trị lớn nhất của:...

Đã gửi bởi quangnghia on 17-02-2016 - 09:44 trong Bất đẳng thức và cực trị

 Cho 2 số x,y nguyên dương thỏa mãn: xy = 1. Tính giá trị lớn nhất của:

Ta có $\frac{x}{x^{4}+y^{2}}\leq \frac{x}{2x^{2}y}\leq \frac{1}{2xy}\leq \frac{1}{2}$

Chứng minh tương tự ta có $\frac{y}{y^{4}+x^{2}}\leq \frac{1}{2}$

$\Rightarrow \sum \frac{x}{x^{4}+y^{2}}\leq 1$




#615509 Tìm GTNN của A= $\frac{(x+y+z)(x+y)}{xyzt}$

Đã gửi bởi quangnghia on 17-02-2016 - 09:40 trong Bất đẳng thức và cực trị

Bài 3 : Cho x,y,z,t >0, x+y+z+t=2. Tìm GTNN của 

A=  $\frac{(x+y+z)(x+y)}{xyzt}$

Ta có $4A=\frac{(x+y+z+t)^{2}(x+y+z)(x+y)}{xyzt}$

$\geq \frac{4(x+y+z)t(x+y+z)(x+y)}{xyzt}$

$\geq \frac{4(x+y+z)^{2}(x+y)}{xyz}\geq \frac{4.4(x+y)z(x+y)}{xyz}$

$\geq \frac{16(x+y)^{2}}{xy}\geq \frac{16.4xy}{xy}\geq 64$

$\Rightarrow A\geq 16$




#615508 Tìm GTNN của A= $\frac{(x+y+z)(x+y)}{xyzt}$

Đã gửi bởi quangnghia on 17-02-2016 - 09:35 trong Bất đẳng thức và cực trị

Bài 4: Tìm GTNN của 

P=$\frac{(x^{3}+y^{3})-(x^{2}+y^{2})}{(x-1)(y-1)}$

Với $x,y\in \mathbb{R}; x,y> 1$

Ta có $\frac{(x^{3}+y^{3})-(x^{2}+y^{2})}{(x-1)(y-1)}$

$= \frac{x^{2}(x-1)+y^{2}(y-1)}{(x-1)(y-1)}=\frac{x^{2}}{y-1}+\frac{y^{2}}{x-1}$

Ta có $\frac{x^{2}}{y-1}+4(y-1)\geq 4x$

$\frac{y^{2}}{x-1}+4(x-1)\geq 4y$

Cộng theo vế ta có $\sum \frac{x^{2}}{y-1}\geq 8$




#615507 Tìm GTNN của A= $\frac{(x+y+z)(x+y)}{xyzt}$

Đã gửi bởi quangnghia on 17-02-2016 - 09:30 trong Bất đẳng thức và cực trị

Bài 2:Cho x,y,z>0. Tìm GTNN của

P=$\frac{x^{2}}{x^{2}+2yz} + \frac{y^{2}}{y^{2}+2zx}+\frac{z^{2}}{z^{2}+2xy}$

Ta có $\frac{x^{2}}{x^{2}+2yz}\geq \frac{x^{2}}{x^{2}+y^{2}+z^{2}}$

Tương tự ta có $\frac{y^{2}}{y^{2}+2xz}\geq \frac{y^{2}}{x^{2}+y^{2}+z^{2}}$

$\frac{z^{2}}{z^{2}+2xy}\geq \frac{z^{2}}{x^{2}+y^{2}+z^{2}}$

Cộng theo vế 3 bất đẳng thức ta có $\sum \frac{x^{2}}{x^{2}+2yz}\geq \frac{x^{2}+y^{2}+z^{2}}{x^{2}+y^{2}+z^{2}}\geq 1$




#615506 Cho hai đường tròn (O), (O') ở ngoài nhau. Đường thẳng $OO'...

Đã gửi bởi quangnghia on 17-02-2016 - 09:24 trong Hình học

Cho hai đường tròn (O), (O') ở ngoài nhau. Đường thẳng $OO'$ cắt hai đường tròn (O), (O') lần lượt tại các điểm A, B, C, D. Kẻ tiếp tuyến chung ngoài EF của hai đường tròn ( E nằm trên (O), F nằm trên (O') ). Gọi M là giao điểm của AE và DF, N là giao của EB và FC. CMR:

a. $MENF$ là hình chữ nhật

b. MN vuông góc với AD

c. $ME.MA=MF.MD$

a) Do AB là đường kính $\Rightarrow \widehat{AEB}=90^{o}$

Do CD là đường kính $\Rightarrow \widehat{CFD}=90^{o}$

Ta có $EO$ song song $FO'$ nên $\widehat{EOB}+\widehat{CO'F}=180^{o}$

Mà $\widehat{EAB}=\frac{1}{2}\widehat{EOB}, \widehat{CDF}=\frac{1}{2}\widehat{CO'F}$

$\Rightarrow \widehat{EAB}+\widehat{CDF}=90^{o}$

$\Rightarrow \widehat{EMF}=90^{o}$

Vậy MENF là hình chữ nhật

b) $\widehat{NEF}=\widehat{ENM}$ ( do MENF là hình chữ nhật)

$\widehat{NEF}=\widehat{BAE}$ ( cùng chắn cung EB)

Vậy $\widehat{EAB}=\widehat{BNM}$

$\Rightarrow \Delta MEN\sim \Delta MCA$

$\Rightarrow$ MN vuông AD

c) Ta có $ME.MA=MK.MN$ (K là giao của MN, AD)

$MK.MN=MF.MD$

$\Rightarrow ME.MA=MF.MD$




#615453 Chứng minh: $PA^{2}+PB^{2}+PC^{2}+PD^...

Đã gửi bởi quangnghia on 16-02-2016 - 21:29 trong Hình học

Cho đường tròn (O), điểm P bất kì nằm trong nó. Qua P kẻ 2 dây AB, CD bất kì đi qua P và vuông góc tại P.

Chứng minh: $PA^{2}+PB^{2}+PC^{2}+PD^{2}$ không phụ thuộc vào P

Ta có $PA^{2}+PB^{2}+PC^{2}+PD^{2}=PA^{2}+PC^{2}+PD^{2}+PB^{2}=AC^{2}+BD^{2}$

Kẻ đường kính AF.

Ta có AB vuông CD ( giả thuyết)

         AB vuông BF ( do AF là đường kính)

nên DC song song BF

nên CBFD là hình thang cân ( hình thang nội tiếp đường tròn là hình thang cân)

nên CF=BD

Khi đó $AC^{2}+BD^{2}=AC^{2}+CF^{2}=AF^{2}=4R^{2}$

Vậy tổng không phụ thuộc vào P




#615361 $y=\sqrt{x+4}+\sqrt{4-x}-\sqrt{1...

Đã gửi bởi quangnghia on 16-02-2016 - 16:06 trong Bất đẳng thức và cực trị

 

Tìm max,min của hàm số 

$y=\sqrt{x+4}+\sqrt{4-x}-\sqrt{16-x^2}$

Lấy đạo hàm của y

$y'=\frac{1}{2\sqrt{x+4}}-\frac{1}{2\sqrt{4-x}}+\frac{x}{\sqrt{16-x^{2}}}$

$= \frac{\sqrt{4-x}-\sqrt{x+4}+2x}{2\sqrt{(x+4)(4-x)}}$

$y'=0$ $\Rightarrow \sqrt{4-x}-\sqrt{4+x}+2x=0$

$\Rightarrow \sqrt{4-x}-\sqrt{4+x}=(\sqrt{4-x})^{2}-(\sqrt{4+x})^{2}$

$\Rightarrow \sqrt{4-x}-\sqrt{4+x}=(\sqrt{4-x}-\sqrt{4+x})(\sqrt{4-x}+\sqrt{4+x})$

$\Rightarrow \sqrt{4-x}-\sqrt{4+x}=0$ hay $\sqrt{4-x}+\sqrt{4+x}=1$

Từ đây giải ra $x=0$

Vẽ bản biển thiên, dễ thấy y max là $2\sqrt{2}$ khi $x=-4$

                                          y min là 0 khi $x=0$




#613153 $\left ( \frac{12}{5} \right )^{x}+\left ( \fra...

Đã gửi bởi quangnghia on 05-02-2016 - 22:22 trong Bất đẳng thức và cực trị

$\left ( \frac{12}{5} \right )^{x}+\left ( \frac{15}{4} \right )^{x}+\left ( \frac{20}{3} \right )^{x}\geq 3^{x}+4^{x}+5^{x}$

Cái bài bất đẳng thức bé bé xinh xinh bạn ghi bên dưới thì mình làm nó như sau:

Ta có $\sqrt{\frac{a^{3}}{b^{3}}}+\sqrt{\frac{a^{3}}{b^{3}}}+1\geq 3\frac{a}{b}$

$\sqrt{\frac{b^{3}}{c^{3}}}+\sqrt{\frac{b^{3}}{c^{3}}}+1\geq 3\frac{b}{c}$

$\sqrt{\frac{c^{3}}{a^{3}}}+\sqrt{\frac{c^{3}}{a^{3}}}+1\geq 3\frac{c}{a}$

Cộng theo vế ta có

$2(\sqrt{\frac{a^{3}}{b^{3}}}+\sqrt{\frac{b^{3}}{c^{3}}}+\sqrt{\frac{c^{3}}{a^{3}}})+3\geq 3(\frac{a}{b}+\frac{b}{c}+\frac{c}{a})\geq 2(\frac{a}{b}+\frac{b}{c}+\frac{c}{a})+\frac{a}{b}+\frac{b}{c}+\frac{c}{a}\geq 2(\frac{a}{b}+\frac{b}{c}+\frac{c}{a})+3$

Vậy $\sqrt{\frac{a^{3}}{b^{3}}}+\sqrt{\frac{b^{3}}{c^{3}}}+\sqrt{\frac{c^{3}}{a^{3}}}\geq \frac{a}{b}+\frac{b}{c}+\frac{c}{a}$




#597175 Sử dụng hình học 9 để chứng minh công thức lượng giác

Đã gửi bởi quangnghia on 06-11-2015 - 22:39 trong Hình học

Sử dụng hình học 9 chứng mình rằng sin2x = 2sinxcosx

Xét $\Delta ABC$ cân tại A có 2 đường cao BH, AM.

Ta có $sin\widehat{BAC}=2sin\widehat{BAM}cos\widehat{BAM}$

$\Leftrightarrow \frac{BH}{BA}=2.\frac{BM}{BA}.\frac{AM}{AB}$

$\Leftrightarrow BA.BH=2BM.AM$

$\Leftrightarrow BH.AB=BC.AM$ (điều này hiển nhiên)

Vậy ta thu được đpcm




#597015 CM: đường thẳng MN luôn đi qua điểm cố định.

Đã gửi bởi quangnghia on 05-11-2015 - 21:04 trong Hình học phẳng

bạn có thể giải thích rõ hơn chỗ $4\overrightarrow{GA}+\overrightarrow{GB}-2\overrightarrow{GC}$ (G cố định)???

Ta có $4\overrightarrow{GA}+\overrightarrow{GB}-2\overrightarrow{GC}=\overrightarrow{0}$

$\Rightarrow 4\overrightarrow{GA}+\overrightarrow{GA}+\overrightarrow{AB}-2\overrightarrow{GA}-2\overrightarrow{AC}=\overrightarrow{0}=3\overrightarrow{GA}+\overrightarrow{AB}-2\overrightarrow{AC}$

Vậy G cố định




#596853 Tính tổng của các hệ số trong khai triển $(3x-4)^{13}$

Đã gửi bởi quangnghia on 04-11-2015 - 20:27 trong Tổ hợp - Xác suất và thống kê - Số phức

 

Câu 2: Tìm số hạng chứa $x^8$ trong khai triển $(1-x^4-\dfrac{1}{x})^{12}$

 

Trình bày chi tiết cho mình nha mọi người.

Hệ sô tổng quát:$C_{12}^{k}(1-x^{4})^{k}(\frac{-1}{x})^{12-k}=C_{12}^{k}.C_{k}^{l}1^{l}(-x^{4})^{k-l}(\frac{-1}{x})^{12-k}=C_{12}^{k}C_{k}^{l}(-1)^{k-l}(-1)^{12-k}x^{5k-4l-12}$

Chọn k, l nguyên sao cho $5k-4l-12=8, l\leq k, k\leq 12$ , chọn các cặp (k,l) sau đó tính tổng 




#596849 Tính tổng của các hệ số trong khai triển $(3x-4)^{13}$

Đã gửi bởi quangnghia on 04-11-2015 - 20:16 trong Tổ hợp - Xác suất và thống kê - Số phức

Câu 1: Tính tổng của các hệ số trong khai triển $(3x-4)^{13}$

Trình bày chi tiết cho mình nha mọi người.

Khai triển:$(3x-4)^{13}=(3x)^{13}+C_{13}^{12}(3x)^{12}(-4)^{1}+....+C_{13}^{1}.(3x)(-4)^{12}+(-4)^{13}$

Vậy tổng hệ số của khai triển:

$3^{13}+C_{13}^{12}3^{12}(-4)+..........+C_{13}^{1}(-4)^{12}+(-4)^{13}=(3-4)^{13}=-1$




#596831 $\sqrt{a^2+3}+\sqrt{b^2+3}+\sqrt...

Đã gửi bởi quangnghia on 04-11-2015 - 18:07 trong Bất đẳng thức và cực trị

Chứng minh BĐT trên sao vậy bạn

Như hồi mình thi HSG cấp thành phố thì mình bình phương 2 vế lên chứng minh tương đương, không thì có cách dùng vectơ mình không rõ, nhưng tên bất đẳng thức này là MIn cốp ki. bạn có thể tham khảo trên mạng để có bài giải chứng minh cụ thể




#596829 $\sqrt{a^2+3}+\sqrt{b^2+3}+\sqrt...

Đã gửi bởi quangnghia on 04-11-2015 - 17:54 trong Bất đẳng thức và cực trị

Cho các số thực dương $a,b,c$ thỏa mãn $ab+bc+ca=3$. Chứng minh rằng

$$\sqrt{a^2+3}+\sqrt{b^2+3}+\sqrt{c^2+3}\geq a+b+c+3$$

Áp dụng bất đẳng thức sau $\sqrt{a^{2}+b^{2}}+\sqrt{c^{2}+d^{2}}+\sqrt{e^{2}+f^{2}}\geq \sqrt{(a+c+e)^{2}+(b+d+f)^{2}}$ (với, a,b,c,d,e,f không âm)

Ta có $\sum \sqrt{a^{2}+3}\geq \sqrt{(a+b+c)^{2}+(3\sqrt{3})^{3}}$

Ta cần chứng minh

$(a+b+c)^{2}+27\geq (a+b+c+3)^{2}$

Rất tiếc đây là 1 đẳng thức với $ab+bc+ca=3$




#596826 CM: đường thẳng MN luôn đi qua điểm cố định.

Đã gửi bởi quangnghia on 04-11-2015 - 17:45 trong Hình học phẳng

Cho $\Delta ABC$ với M, N là điểm thỏa mãn: $\underset{MN}{\rightarrow}= \underset{4MA}{\rightarrow}+\underset{MB}{\rightarrow}-\underset{2MC}{\rightarrow}$

CM: đường thẳng MN luôn đi qua điểm cố định.

Gọi 1 điểm G sao cho $4\overrightarrow{GA}+\overrightarrow{GB}-2\overrightarrow{GC}=\overrightarrow{0}$ (dễ thấy G cố định)

Ta có $\overrightarrow{MN}=4(\overrightarrow{MG}+\overrightarrow{GA})+\overrightarrow{MG}+\overrightarrow{GB}-2\overrightarrow{MG}-2\overrightarrow{GC}=3\overrightarrow{MG}+\overrightarrow{0}=3\overrightarrow{MG}$

Từ đây ta thấy M, N, G thẳng hàng, mà G có định, nói cách khác, MN luôn đi qua điểm cố định (G)